Mean Value Theorm type question

  • Thread starter Thread starter playboy
  • Start date Start date
  • Tags Tags
    Mean Type Value
Click For Summary
The discussion revolves around a problem requiring the application of the Mean Value Theorem to demonstrate that there exists an S in (0, 1) such that |f''(S)| > 4 for a continuously differentiable function f with specific boundary conditions. Participants express uncertainty about the correct application of the theorem and the implications of continuous differentiability on the existence of a second derivative. One user attempts to derive the necessary conditions but struggles with the rigor of their approach and the choice of functions that meet the problem's criteria. The conversation highlights the need for a suitable example of a function that satisfies the given conditions while maintaining continuous differentiability. Ultimately, the thread reflects the complexities involved in applying theoretical concepts to practical problems in calculus.
playboy

Homework Statement




Let f : R ---> R be a Continuosly differentiable function such that f(0) = 0 , f(1) = 1 and f'(0) = f'(1) = 0 .
Show that their exists an S in (0, 1) such that |f''(S)| > 4 .



Homework Equations




I am unsure if I am even supposed to use the mean value theorm in this problem, but it seems like you have to..

THE MEAN VALUE THEORM:

Let f : [a, b] → R be continuous on the closed interval [a, b], and differentiable on the open interval (a, b). Then there exists some R in (a, b) such that

f(b) − f(a) = f'(R)(b − a)



The Attempt at a Solution





Apply derivatives to the mean value theorm above to get:

f'(b) − f'(a) = f''(S)(b − a)


Now, taking the magnitudes, we get

|f'(b) − f'(a)| < |f''(S)||(b − a)|

Now computing, we get

|f'(1) − f'(0)| < |f''(S)||(1 − 0)|

|0 − 0| < |f''(S)|

0< |f''(S)|


Hence

0< |f''(S)|

so indeed, 4< |f''(S)|


How does this sound?

I am very unsure of this...I don't think it is rigourous enough
 
Physics news on Phys.org
You are only given that f is "continuously differentiable". It does not follow from that that f even has a second derivative at any particular point.

Also, f(b)- f(a) = f'(c)(b-a) is true for specific values of a, b, c, not variables. You can say that, for any x, y in [a, b], f(x)- f(y)= f'(c)(x-y) but then what variable are you going to differentiate with respect to: x or y?

Finally, you can say that, for any x in [a,b], f(x)- f(b)= f'(c)(x-a) and differentiate that with respect to x but then you get f'(x)= f'(c), not what you give.

You can use the mean value theorem to say that there exist c in [0, 1] such that f'(c)= f(1)- f(0)= 1.
 
Last edited by a moderator:
HallsofIvy said:
You are only given that f is "continuously differentiable". It does not follow from that that f even has a second derivative at any particular point.

Also, f(b)- f(a) = f'(c)(b-a) is true for specific values of a, b, c, not variables. You can say that, for any x, y in [a, b], f(x)- f(y)= f'(c)(x-y) but then what variable are you going to differentiate with respect to: x or y?

Finally, you can say that, for any x in [a,b], f(x)- f(b)= f'(c)(x-a) and differentiate that with respect to x but then you get f'(x)= f'(c), not what you give.

You can use the mean value theorem to say that there exist c in [0, 1] such that f'(c)= f(1)- f(0)= 1.

Thank you HallsofIvy... I had the Mean Value Theorm wrong...

Let me start again..

a=0, b=1

for any x in [a,b], f(x)- f(b)= f'(c)(x-a)

hence,

f(x)- f(1)= f'(c)(x-0)

and we take the derivative with respect to x

f'(x)- f'(1)= f''(c)(x-0)

now we take the magnitude

|f'(x)- f'(1)| < |f''(c)||x|

and f'(1)=0

|f'(x)| < |f''(c)||x|


and now i don't know where to go from here..

any help please?
 
I think i know how to solve it...

I need to find a function such that f(0)=0 and f(1)=1 and f'(0)=f'(1)=1.

does:


f(x) = 0 for 0 =< x =< 0.25

f(x) = x for 0.25 < x =<0.75

f(x) = 1 for 0.75< x =< 1


but off course, f'(x) won't be conintuously differentiable...

thus, for

|f'(x)| < |f''(c)||x|

choose x=0.75

hence,

for x=0.75, f'(x) = 1

1<|f''(c)||0.75|
4/3 < |f''(c)|

but this function dosn't work in my case...

anybody know a few functions such that f(0)=0 and f(1)=1 and f'(0)=f'(1)=1 ?
 
Question: A clock's minute hand has length 4 and its hour hand has length 3. What is the distance between the tips at the moment when it is increasing most rapidly?(Putnam Exam Question) Answer: Making assumption that both the hands moves at constant angular velocities, the answer is ## \sqrt{7} .## But don't you think this assumption is somewhat doubtful and wrong?

Similar threads

  • · Replies 12 ·
Replies
12
Views
1K
Replies
1
Views
2K
Replies
6
Views
2K
  • · Replies 11 ·
Replies
11
Views
2K
Replies
1
Views
2K
  • · Replies 11 ·
Replies
11
Views
2K
  • · Replies 11 ·
Replies
11
Views
1K
  • · Replies 6 ·
Replies
6
Views
2K
Replies
2
Views
2K
  • · Replies 8 ·
Replies
8
Views
1K